Пескин и Шредер: свободное распространение частиц

В Peskin & Schroeder Ch. 2, с. 14, доказывая, что амплитуда распространения NRQM для свободной частицы всюду отлична от нуля, они движутся от

U ( т )   "="   1 ( 2 π ) 3 г 3 п е я ( п 2 / 2 м ) т е я п ( Икс Икс 0 )
к конечному результату:
U ( т )   "="   ( м 2 π я т ) 3 / 2 е я м ( Икс Икс 0 ) 2 / 2 т .

Я не совсем понимаю все промежуточные шаги. При вычислении первого интеграла я сначала положил его в полярных координатах с z вдоль Икс Икс 0 , но затем я в конечном итоге получаю интеграцию по Гауссу, которая выглядит так, как будто она должна быть равна нулю. Как перейти от первого уравнения ко второму?

РЕДАКТИРОВАТЬ:

Следующим шагом, который я делаю после вышеизложенного, является переписывание интеграла:

1 ( 2 π ) 2 0 1 1 г п г потому что θ п 2 е я ( п 2 / 2 м ) т е я п Δ Икс потому что θ

где Δ Икс | Икс Икс 0 | и я сделал интеграцию ф . Отсюда я получаю

1 ( 2 π ) 2 я Δ Икс 0 г п п е я ( п 2 / 2 м ) т ( е я п Δ Икс е я п Δ Икс ) .

Вроде как эта интеграция должна дать 0 , если я где-то не ошибаюсь. Где моя ошибка?

Возможные дубликаты: physics.stackexchange.com/q/352401/2451 , physics.stackexchange.com/q/105045/2451 и ссылки в них.
Спасибо — я видел это, но похоже, что они доходят до сути только перед тем, как начинается мое замешательство, увы...

Ответы (3)

Вы, кажется, думаете, что

1 ( 2 π ) 2 я Δ Икс 0 г п п е я ( п 2 / 2 м ) т ( е я п Δ Икс е я п Δ Икс ) "=" 0

вероятно, потому что экспоненциальные функции как бы сокращаются, но это не так. Обратите внимание, что

е я п Δ Икс е я п Δ Икс "=" 2 я грех ( п Δ Икс )

что означает, что вы хотите вычислить

2 ( 2 π ) 2 Δ Икс 0 г п п е я ( п 2 / 2 м ) т грех ( п Δ Икс )

и совершенно очевидно, что это не исчезнет. На самом деле мы можем немного поработать над вторым членом исходного интеграла.

1 ( 2 π ) 2 я Δ Икс 0 г п п е я ( п 2 / 2 м ) т е я п Δ Икс

заменять п "=" п , то это равно

1 ( 2 π ) 2 я Δ Икс 0 г п п е я ( п 2 / 2 м ) т е я п Δ Икс

так что ваш исходный интеграл просто

1 ( 2 π ) 2 я Δ Икс г п п е я ( п 2 / 2 м ) т е я п Δ Икс "=" 1 ( 2 π ) 2 я Δ Икс я г г Δ Икс г п е я ( п 2 / 2 м ) т е я п Δ Икс

это просто производная от нормального интеграла Гаусса. Используя общую формулу для интегралов Гаусса

е а Икс 2 + б Икс г Икс "=" π а   е б 2 4 а

который RGJ уже предоставил в своем ответе, мы сразу получаем

( м 2 π я т ) 3 / 2 опыт ( я Δ Икс 2 м 2 т )

что является вашим желаемым результатом.

Отлично, большое спасибо. Моя проблема заключалась в том, чтобы получить коэффициент p из интеграла, и я не думал делать это таким образом.
@ gh3 на самом деле это очень полезный прием, называемый дифференцированием под знаком интеграла, и он будет очень полезен для всех видов гауссовых интегралов. Я бы порекомендовал вам проверить это где-нибудь более подробно.

Обратите внимание на интеграл произвольной функции Гаусса,

е а Икс 2 + б Икс г Икс "=" π а   е б 2 4 а

Незначительный комментарий: квалифицировано а > 0 .
Ага, понял. Моя проблема в том, что при выводе я получаю коэффициент п перед гауссианом, все еще интегрируя от отрицательной бесконечности до положительной бесконечности, что означает, что интеграл оценивается как 0 .
Мой ответ основан на вашем коротком.
@Frobenius: спасибо за уточнение моего ответа. Я думал, что этого будет достаточно!

Намекать :

Сделайте гипотезу о том, что интеграл произвольной функции Гаусса (см. ответ RGJ)

(01) е а Икс 2 + б Икс г Икс "=" π а   е ( б 2 / 4 а )
действует для а , б чисто мнимые числа и для нашего случая
(02.1) а "=" я ( т 2 м ) (02.2) б к "=" я ( Икс Икс 0 ) к , к "=" 1 , 2 , 3
Затем
(03.1) + е а п 1 2 + б 1 п 1 г п 1 "=" π а е ( б 1 2 / 4 а ) "=" 2 π м я т е я м | ( Икс Икс 0 ) 1 | 2 / 2 т (03.2) + е а п 2 2 + б 2 п 2 г п 2 "=" π а е ( б 2 2 / 4 а ) "=" 2 π м я т е я м | ( Икс Икс 0 ) 2 | 2 / 2 т (03.3) + е а п 3 2 + б 3 п 3 г п 3 "=" π а е ( б 3 2 / 4 а ) "=" 2 π м я т е я м | ( Икс Икс 0 ) 3 | 2 / 2 т
Умножая вышеприведенные 3 уравнения рядом, мы имеем
(04) + + + е а п 2 + б п г п 1 г п 2 г п 3 "=" ( π а ) 3 / 2 е ( б 2 / 4 а ) "=" ( 2 π м я т ) 3 / 2 е я м Икс Икс 0 2 / 2 т
или
(05) р 3 е я ( п 2 / 2 м ) т + я п ( Икс Икс 0 ) г 3 п "=" ( 2 π м я т ) 3 / 2 е я м Икс Икс 0 2 / 2 т
и
(06) 1 ( 2 π ) 3 р 3 е я ( п 2 / 2 м ) т + я п ( Икс Икс 0 ) г 3 п "=" ( м 2 π я т ) 3 / 2 е я м Икс Икс 0 2 / 2 т
Итак, попробуйте доказать гипотезу .